Found problems: 110
1980 Putnam, B4
Let $A_1 , A_2 ,\ldots, A_{1066}$ be subsets of a finite set $X$ such that $|A_i | > \frac{1}{2} |X|$ for $1\leq i \leq 1066.$ Prove that there exist ten elements $x_1 ,x_2 ,\ldots , x_{10}$ of $X$ such that every $A_i $ contains at least one of $x_1 , x_2 ,\ldots, x_{10}.$
2004 South East Mathematical Olympiad, 7
A tournament is held among $n$ teams, following such rules:
a) every team plays all others once at home and once away.(i.e. double round-robin schedule)
b) each team may participate in several away games in a week(from Sunday to Saturday).
c) there is no away game arrangement for a team, if it has a home game in the same week.
If the tournament finishes in 4 weeks, determine the maximum value of $n$.
2021 Iran Team Selection Test, 6
Prove that we can color every subset with $n$ element of a set with $3n$ elements with $8$ colors . In such a way that there are no $3$ subsets $A,B,C$ with the same color where :
$$|A \cap B| \le 1,|A \cap C| \le 1,|B \cap C| \le 1$$
Proposed by [i]Morteza Saghafian[/i] and [i]Amir Jafari[/i]
2020 Harvest Math Invitational Team Round Problems, HMI Team #2
2. Let $A$ be a set of $2020$ distinct real numbers. Call a number [i]scarily epic[/i] if it can be expressed as the product of two (not necessarily distinct) numbers from $A$. What is the minimum possible number of distinct [i]scarily epic[/i] numbers?
[i]Proposed by Monkey_king1[/i]
2013 Balkan MO Shortlist, C1
In a mathematical competition, some competitors are friends; friendship is mutual, that is, when $A$ is a friend of $B$, then $B$ is also a friend of $A$.
We say that $n \geq 3$ different competitors $A_1, A_2, \ldots, A_n$ form a [i]weakly-friendly cycle [/i]if $A_i$ is not a friend of $A_{i+1}$ for $1 \leq i \leq n$ (where $A_{n+1} = A_1$), and there are no other pairs of non-friends among the components of the cycle.
The following property is satisfied:
"for every competitor $C$ and every weakly-friendly cycle $\mathcal{S}$ of competitors not including $C$, the set of competitors $D$ in $\mathcal{S}$ which are not friends of $C$ has at most one element"
Prove that all competitors of this mathematical competition can be arranged into three rooms, such that every two competitors in the same room are friends.
([i]Serbia[/i])
2008 Miklós Schweitzer, 2
Let $t\ge 3$ be an integer, and for $1\le i <j\le t$ let $A_{ij}=A_{ji}$ be an arbitrary subset of an $n$-element set $X$. Prove that there exist $1\le i < j\le t$ for which
$$\left| \left( X\,\backslash\, A_{ij}\right) \cup \bigcup_{k\neq i,j}\left( A_{ik}\cap A_{jk}\right) \right| \ge \frac{t-2}{2t-2}n$$
(translated by Miklós Maróti)
2010 Contests, 3
Let $I_1, I_2, I_3$ be three open intervals of $\mathbb{R}$ such that none is contained in another. If $I_1\cap I_2 \cap I_3$ is non-empty, then show that at least one of these intervals is contained in the union of the other two.
2015 Polish MO Finals, 3
Find the biggest natural number $m$ that has the following property: among any five 500-element subsets of $\{ 1,2,\dots, 1000\}$ there exist two sets, whose intersection contains at least $m$ numbers.
1985 Traian Lălescu, 2.3
Let $ X $ be the power set of set of $ \{ 0\}\cup\mathbb{N} , $ and let be a function $ d:X^2\longrightarrow\mathbb{R} $ defined as
$$ d(U,V)=\sum_{n\in\mathbb{N}}\frac{\chi_U (n) +\chi_V (n) -2\chi_{U\cap V} (n)}{2} , $$
where $ \chi_W (n)=\left\{ \begin{matrix} 1,& n\in W\\ 0,& n\not\in W \end{matrix} \right. ,\quad\forall W\in X,\forall n\in\mathbb{N} . $
[b]a)[/b] Prove that there exists an unique $ V' $ such that $ \lim_{k\to\infty} d\left( \{ k+i|i\in\mathbb{N}\} , V'\right) =0. $
[b]b)[/b] Demonstrate that for all $ V\in X $ there exists a $ v\in\mathbb{N} $ with $ d\left( \left\{ \frac{3}{2} -\frac{1}{2}(-1)^{v} \right\} , V \right) >\frac{1}{k} . $
[b]c)[/b] Let $ f: X\longrightarrow X,\quad f(X)=\left\{ 1+x|x\in X\right\} . $ Calculate $ d\left( f(A),f(B) \right) $ in terms of $ d(A,B) $ and prove that $ f $ admits an unique fixed point.
Russian TST 2014, P1
Given are twenty-two different five-element sets, such that any two of them have exactly two elements in common. Prove that they all have two elements in common.
2019 All-Russian Olympiad, 7
$24$ students attend a mathematical circle. For any team consisting of $6$ students, the teacher considers it to be either [b]GOOD [/b] or [b]OK[/b]. For the tournament of mathematical battles, the teacher wants to partition all the students into $4$ teams of $6$ students each. May it happen that every such partition contains either $3$ [b]GOOD[/b] teams or exactly one [b]GOOD[/b] team and both options are present?
1989 Putnam, B4
Can a countably infinite set have an uncountable collection of non-empty subsets such that the intersection of any two of them is finite?
2017 Romanian Master of Mathematics, 3
Let $n$ be an integer greater than $1$ and let $X$ be an $n$-element set. A non-empty collection of subsets $A_1, ..., A_k$ of $X$ is tight if the union $A_1 \cup \cdots \cup A_k$ is a proper subset of $X$ and no element of $X$ lies in exactly one of the $A_i$s. Find the largest cardinality of a collection of proper non-empty subsets of $X$, no non-empty subcollection of which is tight.
[i]Note[/i]. A subset $A$ of $X$ is proper if $A\neq X$. The sets in a collection are assumed to be distinct. The whole collection is assumed to be a subcollection.
2017 Romanian Masters In Mathematics, 3
Let $n$ be an integer greater than $1$ and let $X$ be an $n$-element set. A non-empty collection of subsets $A_1, ..., A_k$ of $X$ is tight if the union $A_1 \cup \cdots \cup A_k$ is a proper subset of $X$ and no element of $X$ lies in exactly one of the $A_i$s. Find the largest cardinality of a collection of proper non-empty subsets of $X$, no non-empty subcollection of which is tight.
[i]Note[/i]. A subset $A$ of $X$ is proper if $A\neq X$. The sets in a collection are assumed to be distinct. The whole collection is assumed to be a subcollection.
2011 IFYM, Sozopol, 4
For each subset $S$ of $\mathbb{N}$, with $r_S (n)$ we denote the number of ordered pairs $(a,b)$, $a,b\in S$, $a\neq b$, for which $a+b=n$. Prove that $\mathbb{N}$ can be partitioned into two subsets $A$ and $B$, so that $r_A(n)=r_B(n)$ for $\forall$ $n\in \mathbb{N}$.
2009 Serbia National Math Olympiad, 3
Determine the largest positive integer $n$ for which there exist pairwise different sets $\mathbb{S}_1 , ..., \mathbb{S}_n$ with the following properties:
$1$) $|\mathbb{S}_i \cup \mathbb{S}_j | \leq 2004$ for any two indices $1 \leq i, j\leq n$, and
$2$) $\mathbb{S}_i \cup \mathbb{S}_j \cup \mathbb{S}_k = \{ 1,2,...,2008 \}$ for any $1 \leq i < j < k \leq n$
[i]Proposed by Ivan Matic[/i]
2009 Miklós Schweitzer, 3
Prove that there exist positive constants $ c$ and $ n_0$ with the following property. If $ A$ is a finite set of integers, $ |A| \equal{} n > n_0$, then
\[ |A \minus{} A| \minus{} |A \plus{} A| \leq n^2 \minus{} c n^{8/5}.\]
2022 Vietnam TST, 6
Given a set $A=\{1;2;...;4044\}$. They color $2022$ numbers of them by white and the rest of them by black. With each $i\in A$, called the [b][i]important number[/i][/b] of $i$ be the number of all white numbers smaller than $i$ and black numbers larger than $i$. With every natural number $m$, find all positive integers $k$ that exist a way to color the numbers that can get $k$ important numbers equal to $m$.
2001 VJIMC, Problem 4
Let $A,B,C$ be nonempty sets in $\mathbb R^n$. Suppose that $A$ is bounded, $C$ is closed and convex, and $A+B\subseteq A+C$. Prove that
$B\subseteq C$.
Recall that $E+F=\{e+f:e\in E,f\in F\}$ and $D\subseteq\mathbb R^n$ is convex iff $tx+(1-t)y\in D$ for all $x,y\in D$ and any $t\in[0,1]$.
2000 VJIMC, Problem 1
Is there a countable set $Y$ and an uncountable family $\mathcal F$ of its subsets such that for every two distinct $A,B\in\mathcal F$, their intersection $A\cap B$ is finite?
2023 Miklós Schweitzer, 1
Prove that if $X{}$ is an infinite set of cardinality $\kappa$ then there is a collection $\mathcal{F}$ of subsets of $X$ such that[list]
[*]For any $A\subseteq X$ with cardinality $\kappa$ there exists $F\in\mathcal{F}$ for which $A\cap F$ has cardinality $\kappa,$ and
[*]$X$ cannot be written as the union of less than $\kappa$ sets from $\mathcal{F}$ and a single set of cardinality less than $\kappa$.
[/list]
2023 Vietnam National Olympiad, 6
There are $n \geq 2$ classes organized $m \geq 1$ extracurricular groups for students. Every class has students participating in at least one extracurricular group. Every extracurricular group has exactly $a$ classes that the students in this group participate in. For any two extracurricular groups, there are no more than $b$ classes with students participating in both groups simultaneously.
a) Find $m$ when $n = 8, a = 4 , b = 1$ .
b) Prove that $n \geq 20$ when $m = 6 , a = 10 , b = 4$.
c) Find the minimum value of $n$ when $m = 20 , a = 4 , b = 1$.
2017 Polish MO Finals, 4
Prove that the set of positive integers $\mathbb Z^+$ can be represented as a sum of five pairwise distinct subsets with the following property: each $5$-tuple of numbers of form $(n,2n,3n,4n,5n)$, where $n\in\mathbb Z^+$, contains exactly one number from each of these five subsets.
1990 China Team Selection Test, 4
There are arbitrary 7 points in the plane. Circles are drawn through every 4 possible concyclic points. Find the maximum number of circles that can be drawn.
1978 Romania Team Selection Test, 3
Let $ p $ be a natural number and let two partitions $ \mathcal{A} =\left\{ A_1,A_2,...,A_p\right\} ,\mathcal{B}=\left\{ B_1,B_2,...B_p\right\} $ of a finite set $ \mathcal{M} . $ Knowing that, whenever an element of $ \mathcal{A} $ doesn´t have any elements in common with another of $ \mathcal{B} , $ it holds that the number of elements of these two is greater than $ p, $ prove that $ \big| \mathcal{M}\big|\ge\frac{1}{2}\left( 1+p^2\right) . $ Can equality hold?